Simplest Or Nicest Proof That $1+x \le E^x - Math Stack Exchange
Có thể bạn quan tâm
-
- Home
- Questions
- Tags
- Users
- Unanswered
- Teams
Ask questions, find answers and collaborate at work with Stack Overflow for Teams.
Try Teams for free Explore Teams - Teams
-
Ask questions, find answers and collaborate at work with Stack Overflow for Teams. Explore Teams
Teams
Q&A for work
Connect and share knowledge within a single location that is structured and easy to search.
Learn more about Teams Simplest or nicest proof that $1+x \le e^x$ Ask Question Asked 11 years, 3 months ago Modified 2 years, 7 months ago Viewed 94k times 128 $\begingroup$The elementary but very useful inequality that $1+x \le e^x$ for all real $x$ has a number of different proofs, some of which can be found online. But is there a particularly slick, intuitive or canonical proof? I would ideally like a proof which fits into a few lines, is accessible to students with limited calculus experience, and does not involve too much analysis of different cases.
Share Cite Follow edited Feb 17, 2018 at 10:41 Parcly Taxel 105k20 gold badges116 silver badges202 bronze badges asked Sep 25, 2013 at 10:04 Ashley MontanaroAshley Montanaro 1,3972 gold badges9 silver badges8 bronze badges $\endgroup$ 3- 2 $\begingroup$ Question about the same inequality: Prove that $e^x\ge x+1$ for all real $x$ $\endgroup$ – Martin Sleziak Commented Sep 25, 2013 at 12:31
- 2 $\begingroup$ There are many great answers below. I now wonder whether, as a question with no objective right answer, this should be community wiki? $\endgroup$ – Ashley Montanaro Commented Sep 26, 2013 at 21:47
- $\begingroup$ This is a duplicate of this : math.stackexchange.com/questions/252541/… $\endgroup$ – Guy Fsone Commented Nov 10, 2017 at 18:30
26 Answers
Sorted by: Reset to default Highest score (default) Date modified (newest first) Date created (oldest first) 181 $\begingroup$Another way (not sure if its "simple" though!): $y = x+1$ is the tangent line to $y = e^x$ when $x= 0$. Since $e^x$ is convex, it always remains above its tangent lines.
Share Cite Follow answered Sep 25, 2013 at 10:23 MacavityMacavity 47.5k6 gold badges38 silver badges72 bronze badges $\endgroup$ 8- 15 $\begingroup$ That's my favourite argument. I'm afraid it fails on the "is accessible to students with limited calculus experience" criterion, but it's boootiful. $\endgroup$ – Daniel Fischer Commented Sep 25, 2013 at 12:28
- 19 $\begingroup$ @DanielFischer: Are you sure that it fails? Tangent lines can appear much earlier then the derivatives and it have a nice graphical representation (sure, representation might not be mathematically precise). $\endgroup$ – Maja Piechotka Commented Sep 25, 2013 at 13:38
- 4 $\begingroup$ @Maciej I'm not sure. But without some calculus to build on, I think it would be too hand-wavy for me. $\endgroup$ – Daniel Fischer Commented Sep 25, 2013 at 13:44
- 20 $\begingroup$ The convexity, at least, is elementary: $e^{x+z} - 2 e^x + e^{x-z} = (e^z-1)^2 e^{x-z} > 0$ for all real $x,z$, with equality iff $z=0$. $\endgroup$ – Noam D. Elkies Commented Sep 25, 2013 at 14:55
- 30 $\begingroup$ @NoamD.Elkies -- for large values of "elementary". $\endgroup$ – Michael Lorton Commented Sep 25, 2013 at 18:52
$$ e^x = \lim_{n\to\infty}\left(1+\frac xn\right)^n\ge1+x $$
by Bernoulli's inequality.
Share Cite Follow answered Sep 25, 2013 at 14:21 John GowersJohn Gowers 25.4k6 gold badges66 silver badges102 bronze badges $\endgroup$ 1- 2 $\begingroup$ This is what I thought of when I saw this question. (+1) $\endgroup$ – robjohn ♦ Commented Sep 27, 2013 at 0:31
The shortest proof I could think of: $$1 + x \leq 1 + x + \frac{x^2}{2!} + \frac{x^3}{3!} + \cdots = e^x.$$
However, it is not completely obvious for negative $x$.
Using derivatives:
Take $f(x) = e^x - 1 - x$. Then $f'(x) = e^x - 1$ with $f'(x) = 0$ if and only if $x = 0$. But this is a minimum (global in this case) since $f''(0) = 1 > 0$ (the second derivative test). So $f(x) \geq 0$ for all real $x$, and the result follows.
Another fairly simple proof (but it uses Newton's generalization of the Binomial Theorem which is often covered in precalculus):
We proceed by contradiction. Suppose the inequality does not hold, i.e., $e^x < 1 + x$ for some $x$. Then $e^{kx} < (1 + x)^k$. Now set $x = 1/k$ so that \begin{align*} e &< \left( 1 + \frac{1}{k} \right)^k\\ &= 1 + \frac{k}{1}\left( \frac{1}{k} \right)^1 + \frac{k(k - 1)}{1 \cdot 2}\left( \frac{1}{k} \right)^2 + \frac{k(k - 1)(k - 2)}{1 \cdot 2 \cdot 3}\left( \frac{1}{k} \right)^3 + \cdots\\ &< 1 + \frac{k}{1}\left( \frac{1}{k} \right)^1 + \frac{k^2}{1 \cdot 2}\left( \frac{1}{k} \right)^2 + \frac{k^3}{1 \cdot 2 \cdot 3}\left( \frac{1}{k} \right)^3 + \cdots\\ &= 1 + 1 + \frac{1}{2!} + \frac{1}{3!} + \cdots\\ &= e, \end{align*} which is absurd. Therefore $1 + x \leq e^x$ for all real $x$.
By the way, this is where $$e = \lim_{k \to \infty}\left( 1 + \frac{1}{k} \right)^k$$ comes from because $$\lim_{k \to \infty}\frac{k(k - 1)}{k^2} = \lim_{k \to \infty}\frac{k(k - 1)(k - 2)}{k^3} = \cdots = \lim_{k \to \infty}\frac{k(k - 1)(k - 2) \cdots (k - n)}{k^{n + 1}} = \cdots = 1.$$
Share Cite Follow edited Sep 26, 2013 at 17:49 answered Sep 25, 2013 at 10:13 glebovgglebovg 10.3k2 gold badges35 silver badges58 bronze badges $\endgroup$ 5- 23 $\begingroup$ Is it that evident for negative $x$? $\endgroup$ – Macavity Commented Sep 25, 2013 at 10:16
- 4 $\begingroup$ @Macavity for x<-1 is obvious $\endgroup$ – BЈовић Commented Sep 25, 2013 at 17:35
- 1 $\begingroup$ In your third proof : Why should such a $x$ be of the form $1/k$ with $k$ a positive integer ? $\endgroup$ – user37238 Commented Sep 26, 2013 at 15:20
- 1 $\begingroup$ @user37238 Here, $k$ is not necessarily an integer because, technically, the proof uses the Binomial Series. This explains why I wrote the binomial coefficients using the falling factorial. $\endgroup$ – glebovg Commented Sep 26, 2013 at 17:58
- $\begingroup$ I'm confused: you assume (for a contradiction) that $e^{x}<1+x$ for some real $x$, but then you claim that you can set $x=1/k$ without further justification. Would you not need to prove that $1/k$ is a valid choice of $x$ under such an assumption? $\endgroup$ – Will R Commented Jul 28, 2015 at 8:52
Let $f(x) = e^x-(1+x)$, then $f^\prime(x) = e^x-1$. Hence $f^\prime(x)=0$ iff $x=0$. Furthermore $f^{\prime\prime}(0) = e^0=1>0$, thus $f(0)=0$ must be the global minimum of $f$, proving your claim.
Share Cite Follow answered Sep 25, 2013 at 10:14 AbelAbel 7,39219 silver badges22 bronze badges $\endgroup$ 1- 2 $\begingroup$ I like the idea behind this proof : just study a function and show that this function is always non-negative. But I think that it could be a little simpler if you say that $f'\ge 0$ on $[0,+\infty($ (and consequently $f$ increases on this interval) and $f'\le 0$ on $)-\infty,0]$ (and $f$ decreases on this interval) so $f(0)=0$ is a global minimum of $f$. $\endgroup$ – user37238 Commented Sep 25, 2013 at 14:59
One that's not been mentioned so far(?): knowing that $$ 0 < e^x = 1 + x + \frac{x^2}{2} + \frac{x^3}{3!} + \frac{x^4}{4!} + \frac{x^5}{5!} + \cdots $$ proves the inequality except for $-1 < x < 0$. But in that region $$ e^x - (1+x) = \frac{x^2}{2} + \frac{x^3}{3!} + \frac{x^4}{4!} + \frac{x^5}{5!} + \cdots $$ is an alternating series whose terms decrease in absolute value and start out positive. Therefore it is positive by the usual argument: group the terms as $$ e^x - (1+x) = \left( \frac{x^2}{2} + \frac{x^3}{3!} \right) + \left( \frac{x^4}{4!} + \frac{x^5}{5!} \right) + \cdots $$ and observe that each combined term is positive, QED.
(This actually works for $-3 < x < 0$, but you still want to use $e^x > 0$ to prove the inequality for very negative $x$.)
Share Cite Follow edited Sep 26, 2013 at 20:51 answered Sep 25, 2013 at 14:54 Noam D. ElkiesNoam D. Elkies 26.2k1 gold badge67 silver badges83 bronze badges $\endgroup$ 2- 2 $\begingroup$ Why does knowing that $0<e^x$ give the result for $x\le-1$? $\endgroup$ – John Gowers Commented Sep 25, 2013 at 14:56
- 15 $\begingroup$ $x \leq -1$ means $x+1 \leq 0 < e^x$. $\endgroup$ – Noam D. Elkies Commented Sep 25, 2013 at 14:58
Repeatedly using $1 + x \le \left(1 + \frac{x}{2} \right)^2$, we have \begin{align} 1 + x \le \left(1 + \frac x 2\right)^2 \le \left(1 + \frac x 4\right)^4 \le \left(1 + \frac x 8\right)^8 \le \dots \le \left(1 + \frac x {2^k}\right)^{2^k}. \end{align} Taking the limit of $k \rightarrow \infty$ yields $$ 1 + x \le e^x. \qquad\qquad(1) $$
Another proof using the technique in this post. By the AM-GM inequality, $$ \sqrt[n]{1 \times \cdots \times 1 \times (1 + x)} \le \frac{1 + \dots + 1 + (1 + x)}{n} =1 + \frac{x}{n}. $$ So, $$ 1+x \le \left(1 + \frac{x}{n} \right)^n. $$ Taking the limit of $n \rightarrow \infty$ yields (1).
Share Cite Follow edited Apr 13, 2017 at 12:21 community wiki 3 revshbp $\endgroup$ Add a comment | 21 $\begingroup$If $x \ge 0$ then $$\begin{align} e^x = 1 + \int_0^x e^t\,\mathrm dt &= 1 + \int_0^x\left( 1 + \int_0^t e^u \,\mathrm du\right)\,\mathrm dt \\&= 1+x + \int_0^x \int_0^t e^u\,\mathrm du\,\mathrm dt \ge 1+x\end{align}$$
If $x \le 0$ then $$\begin{align}e^x = 1 - \int_x^0 e^t\,\mathrm dt &= 1 - \int_x^0\left( 1 - \int_t^0 e^u\,\mathrm du\right)\,\mathrm dt \\ &= 1+x + \int_x^0 \int_t^0 e^u\,\mathrm du\,\mathrm dt \ge 1+x\end{align}$$
Share Cite Follow edited Sep 26, 2013 at 12:12 filmor 5512 silver badges9 bronze badges answered Sep 25, 2013 at 10:45 merciomercio 50.9k2 gold badges83 silver badges133 bronze badges $\endgroup$ Add a comment | 14 $\begingroup$For completeness, using $\exp(x)=1+x+\frac{1}{2}x^2+\dots$, the inequality is trivial for $x\ge 0$. It is also trivial for $x<-1$.
It remains to show the case $-1<x<0$. Replacing $x$ by $-x$, one need to show $1-x < e^{-x}$ for $0<x<1$, or $$1+x+\frac{1}{2}x^2+\dots=e^x <\frac{1}{1-x}=1+x+x^2+\dots,$$ we are done.
Share Cite Follow edited Jun 23, 2019 at 13:12 J. W. Tanner 63k4 gold badges42 silver badges88 bronze badges answered Sep 25, 2013 at 11:41 Ma MingMa Ming 7,50220 silver badges35 bronze badges $\endgroup$ 3- $\begingroup$ @Dr.MV $x+1<0< e^x$ $\endgroup$ – Ma Ming Commented Nov 21, 2016 at 2:47
- $\begingroup$ Yes, of course. From the post, it seemed that you were inferring this from the series expansion. $\endgroup$ – Mark Viola Commented Nov 21, 2016 at 3:24
- $\begingroup$ Did you mean it's trivial for $x \mathbf \le -1$ ? $\endgroup$ – J. W. Tanner Commented Mar 14, 2019 at 0:31
Beautiful answers, but nobody used The Mean Value Theorem. Apply MVT on $[0,x] $ for $x>0$. There is some $c\in (0,x)$ such that:
\begin{align} \frac{e^x-e^0}{x-0} = e^c > 1 \end{align} So \begin{align} e^x>1+x \end{align} Something similar can be done for $x<0$. Finally note that we have equality when $x=0$. So we get the desired result: \begin{align} e^x\geq 1+x \end{align}
Share Cite Follow answered Nov 10, 2017 at 19:57 ShashiShashi 8,8981 gold badge14 silver badges39 bronze badges $\endgroup$ Add a comment | 10 $\begingroup$Completing glebovg's answer :
the inequality $1+x \le e^x$ clearly holds for $x \leq -1$,
suppose $x \geq -1$ :
the series $e^x = 1 + x + \frac{x^2}{2!} + \frac{x^3}{3!} + \dots$ can be written (grouping the terms in pair) :
$$e^x = 1 + x + \sum_{k \geq 1} \left( \frac{x^{2k}}{(2k)!} + \frac{x^{2k+1}}{(2k+1)!} \right)$$
$$e^x = 1 + x + \sum_{k \geq 1} x^{2k}\left( \frac{1}{(2k)!} + \frac{x}{(2k+1)!} \right)$$
$$e^x = 1 + x + \sum_{k \geq 1} x^{2k}\left( \frac{2k + 1 + x}{(2k+1)!} \right)$$
under the assumption $x \geq -1$, the $\sum$ part is clearly a sum of positive numbers.
Share Cite Follow edited Apr 13, 2017 at 12:19 CommunityBot 1 answered Sep 25, 2013 at 14:58 LeGECLeGEC 2011 silver badge3 bronze badges $\endgroup$ Add a comment | 7 $\begingroup$We want to prove that $1+x\le e^x$ for any $x\in\mathbb R$. Setting $x=\log(u)$, this is equivalent to proving:
$$ 1+\log(u)\le u $$ for any $u\in (0, \infty)$.
This is true because:
$$ 1+\log(u)=1+\int_1^u\frac1tdt\le1+\int_1^u1dt=1+u-1=u $$
Some care is needed to establish that the inequality is true for both $u\ge1$ and $0<u\le1$. In the second case, we can see this more clearly by writing:
$$ 1+\int_1^u\frac1tdt=1+\int_u^1-\frac1tdt\le1+\int_u^1-1dt=1-1+u=u $$
Share Cite Follow edited Sep 26, 2013 at 11:55 answered Sep 25, 2013 at 15:20 John GowersJohn Gowers 25.4k6 gold badges66 silver badges102 bronze badges $\endgroup$ Add a comment | 6 $\begingroup$There is an amusing proof that I found yesterday that $e^x>x$ for every $x\in \mathbb{R}$. It is obvious that $e^x>x$ if $x<0$ since the LHS is positive and the RHS is negative. Suppose that for some $a\ge 0$, the inequality $e^a\le a$ holds. Then $a\ge e^a\ge 1$ since $e^a\ge e^0$ because $a\ge 0$. But now we can see that $a\ge 1$ and again, $a\ge e^a\ge e^1$ and so $a\ge e$. We continue applying the same observation and conclude that $a\ge e^{^{e}}$ and so on, which means that $a$ is unbounded which is a contradiction.
Share Cite Follow edited Dec 6, 2020 at 17:35 answered May 21, 2019 at 9:57 Konstantinos GaitanasKonstantinos Gaitanas 9,1934 gold badges31 silver badges47 bronze badges $\endgroup$ 1- 1 $\begingroup$ It's actually a great proof $\endgroup$ – emandret Commented Jan 17, 2021 at 0:24
For positive values of $ x $ We can use the following characterization of $e^x$ $$ e^x=\lim_{t\to \infty} \Big( 1+\frac{1}{t}\Big)^{tx},\quad t> 0,x\geq 0. $$ The Bernoulli's inequality states that $(1 + y)^r \geq 1 + ry$ for every $r \geq 1$ and every real number $y \geq −1$. Then for $y=\frac{1}{t}$ and $t>0$ such that $r=tx\geq 1$ we have \begin{align} e^x= &\lim_{t\to \infty} \Big( 1+\frac{1}{t}\Big)^{tx}\\ \geq &\lim_{t\to \infty} \Big(1+\frac{1}{t}(tx) \Big)\\ = & 1+x \end{align}
Share Cite Follow edited Dec 6, 2021 at 12:09 answered Sep 25, 2013 at 14:08 Elias CostaElias Costa 15.1k5 gold badges55 silver badges91 bronze badges $\endgroup$ 2- $\begingroup$ I just came across this answer 8 years later haha. However, I believe that $(1+y)^r\geq 1+ry$ does not hold for $0 < r < 1$. For example, $(1+3)^{0.5} = 2$ while $1+3\cdot 0.5 = 2.5$. $\endgroup$ – Gareth Ma Commented Dec 6, 2021 at 8:30
- $\begingroup$ @GarethMa Yes, you are correct. The inequality is valid for $r \geq 1$. I will make the correction. $\endgroup$ – Elias Costa Commented Dec 6, 2021 at 12:08
For $x>0$ we have $e^t>1$ for $0<t<x$
Hence, $$x=\int_0^x1dt \color{red}{\le} \int_0^xe^tdt =e^x-1 \implies 1+x\le e^x$$ For $x<0$ we have $e^{t} <1$ for $x <t<0$
$$-x=\int^0_x1dt \color{red}{\ge} \int^0_xe^tdt =1-e^x \implies 1+x\le e^x$$
Share Cite Follow edited Nov 10, 2017 at 18:38 answered Nov 10, 2017 at 18:28 Guy FsoneGuy Fsone 24.6k5 gold badges65 silver badges108 bronze badges $\endgroup$ Add a comment | 4 $\begingroup$One which uses $\exp(x) = \frac 1{\exp(-x)} $ $$ 1 + x \underset{ \text{obvious}\\ \text{for $x>0$}}{\lt} 1 + x + {x^2 \over 2!} + {x^3 \over 3!} + ... = {1 \over 1 - x + {x^2 \over 2!} - { x^31 \over 3!} + ... } \tag 1 $$ Now we replace $+x$ by its negative counterparts and get similarily $$ 1 - x \underset{ \quad \text{for $x>0$}\\ \text{but not obvious}}{\lt} 1 - x + {x^2 \over 2!} - {x^3 \over 3!} + ... = {1 \over 1 + x + {x^2 \over 2!} + {x^3 \over 3!} + ... }\tag 2$$ But now the comparision with the fraction on the rhs becomes obvious if we look at the reciprocals. The reciprocal ${1\over 1-x}=1+x+x^2+x^3+...$ is and we get $$ {1 \over 1 - x} = 1+x+x^2+... \underset{ \text{obvious}\\ \text{for $x>0$}}{\gt} 1 + x + {x^2 \over 2!} + {x^3 \over 3!} + ... = {1 \over 1 - x + {x^2 \over 2!} - {x^3 \over 3!} + ... }\\ \tag 3 $$
Share Cite Follow edited Dec 16, 2021 at 16:47 answered Sep 26, 2013 at 8:52 Gottfried HelmsGottfried Helms 35.3k3 gold badges68 silver badges145 bronze badges $\endgroup$ 1- 1 $\begingroup$ upps, I see there was another answer earlier, but which seemed too short for me to not only skim over it... $\endgroup$ – Gottfried Helms Commented Sep 26, 2013 at 9:28
Proof by induction (works for natural numbers)
Assume it works for n
1 + n < e^nThen we prove that it works for n+1
1 + (n+1) < e^(n+1)Proof
1 + n < e^n or 1 + n + 1 < e^n + 1 or 1 + n + 1 < e^n + e^n since e^n > 1 or 1 + n + 1 < e^n * 2 or 1 + (n+1) < e^n * e since e > 2 or 1 + (n+1) < e^(n+1)hence it is true for n+1 if true for n. We know it is true for 1, hence by induction is true of 2, 3, 4...so on.
Share Cite Follow edited Sep 25, 2013 at 23:19 answered Sep 25, 2013 at 22:41 tihomtihom 1473 bronze badges $\endgroup$ 4- $\begingroup$ This isn't really a proof. Like, where is your inductive step? $\endgroup$ – Alexander Commented Sep 25, 2013 at 23:00
- 1 $\begingroup$ updated the answer works for natural number only though $\endgroup$ – tihom Commented Sep 25, 2013 at 23:20
- $\begingroup$ @tihom You can get a general proof from your proof in the following way: $floor(x)+1>x\geq floor(x) $ where $floor(x)$ is the greatest integer function. Now $$e^x\geq e^{floor(x)}> floor(x)+1> x$$ whenever $x\geq 1$ $\endgroup$ – Sedergine Commented Sep 18, 2021 at 2:07
- $\begingroup$ More precisely, this was for $e^x>x$ but refining your inductive argument to $e^n>n+2$ for $n\geq 2$ we can easily fix it. $\endgroup$ – Sedergine Commented Sep 18, 2021 at 2:34
Another simple proof...
Define function $f(x)=e^x-(x+1)$. The minimum value is $0$ at $x=0$, it's also convex, so $f(x) \ge 0$.
Share Cite Follow answered Feb 1, 2018 at 15:43 karakfakarakfa 2,71515 silver badges15 bronze badges $\endgroup$ Add a comment | 2 $\begingroup$For $x > 0$, consider the mean value theorem on the interval $[0,x]$. Then $$e^x - e^0 \geq \inf_{(0,x)} e^c \cdot (x - 0) = x,$$ implying $e^x \geq 1+x$. For $x < 0$, apply MTV on $[x,0]$: $$e^0 - e^x \leq \sup_{(x,0)} e^c \cdot (0 - x) = -x,$$ giving us $-e^x \leq -x - 1$, or $e^x \geq x + 1$. Then check $x = 0$ and the equality is proven.
Share Cite Follow answered Jul 28, 2015 at 7:28 user217285user217285 5,8151 gold badge18 silver badges31 bronze badges $\endgroup$ Add a comment | 1 $\begingroup$The series expansion of $(1+x)$ is $(1+x)$, while $\exp(x)=1+x+\frac{1}{2}x^2+\frac{1}{6}x^3+...$. Subtracting the second from the first one you have the difference $d=\exp(x)-(1+x)=\frac{1}{2}x^2+\frac{1}{6}x^3+...$ which is zero only for $x=0$ otherwise $d\gt 0$ Q.E.D.
Share Cite Follow answered Sep 25, 2013 at 10:12 Riccardo.AlestraRiccardo.Alestra 10.7k4 gold badges41 silver badges84 bronze badges $\endgroup$ 3- 3 $\begingroup$ Same comment, is it trivially evident for negative $x$ that $d>0$? $\endgroup$ – Macavity Commented Sep 25, 2013 at 10:17
- $\begingroup$ @Macavity: make the substitution: $x\to -y$. Anyway it's not trivial. $\endgroup$ – Riccardo.Alestra Commented Sep 25, 2013 at 10:26
- $\begingroup$ I have seen that proof done considering three regions separately - viz. $x \ge 0, -1 < x < 0, x \le -1$. The first region is trivial, the rest two not so much :( $\endgroup$ – Macavity Commented Sep 25, 2013 at 10:34
Let $f(x)=\exp(x)-x-1$. Then, $f'(0)=0$. But $f$ is strictly convex (a difference of a strictly convex function and an affine one), so that $0$ most be a unique global minimum. Hence, $\exp(x)-x-1=f(x)\geq f(0)=0$ for all $x\in\mathbb{R}$.
Share Cite Follow answered Sep 25, 2013 at 10:35 triple_sectriple_sec 23.5k3 gold badges43 silver badges94 bronze badges $\endgroup$ Add a comment | 1 $\begingroup$We want to show that (1) $$1+x\leq e^x,$$ for $x\in\mathbb{R}$. When $x\geq 0$, we have $$1+x\leq 1+x+\frac{x^2}{2!}+\frac{x^3}{3!}+\cdots=e^x.$$ Suppose $x=-X$, where $X>1$, then $1+x=1-X<0$ and $e^{x}=e^{-X}=1/e^X>0$. Hence (1) holds.
Now take logarithms of (1) to obtain $$\log(1+x)\leq x.$$ But $$\log(1+x) = x-\frac{x^2}{2}+\frac{x^3}{3}-\cdots,$$ where $|x|<1$. Suppose $x=-X$, where $0<X<1$, then $$\log(1+x)=\log(1-X)=-X-\frac{X^2}{2}-\frac{X^3}{3}-\cdots<-X=x.$$ Or, equivalently, $$1+x<e^x,$$ where $-1<x<0$.
Share Cite Follow edited Sep 25, 2013 at 12:59 answered Sep 25, 2013 at 12:18 pshmath0pshmath0 11k6 gold badges48 silver badges88 bronze badges $\endgroup$ 1- 1 $\begingroup$ Third line, I think you meant "Suppose $x=-X$" and not "Suppose $x<-X$". $\endgroup$ – user37238 Commented Sep 25, 2013 at 12:49
The fact $\frac{d}{dx} e^x = e^x$ is nicely demonstrated using the self-similar nature of exponential functions. (See my answer here.)
This justifies (actually, declares) that $y=x+1$ is tangent to $y=e^x$; thereafter, since the slope increases (or decreases) as $x$ gets larger (respectively, smaller) the line and curve cannot meet again (which is an informal way of stating the convexity property).
Share Cite Follow edited Apr 13, 2017 at 12:19 CommunityBot 1 answered Sep 25, 2013 at 23:21 BlueBlue 80.6k14 gold badges124 silver badges247 bronze badges $\endgroup$ Add a comment | 1 $\begingroup$We know the function $x^x$ has a single local minimum at $x=\frac1e$. Thus, for positive $x$, we have: \begin{align} \left(\frac1e\right)^{1/e}&\le x^x\\ e^{1/e}&\ge\frac1{x^x}\\ e^{1/xe}&\ge\frac1x\\ e^{1/xe-1}&\ge\frac1{xe}\\ e^{(1/xe-1)}&\ge\left(\frac1{xe}-1\right)+1 \end{align} Let $t=\frac1{xe}-1$. If $x>0$, we have $t>-1$. Thus, for all $t>-1$: $$e^t\ge t+1$$ (To prove the above for $t\le -1$, simply note that the left-hand side is always positive while the right-hand side would be zero or negative.) QED.
Share Cite Follow answered Oct 29, 2015 at 1:28 Akiva WeinbergerAkiva Weinberger 24.9k2 gold badges49 silver badges122 bronze badges $\endgroup$ Add a comment | 1 $\begingroup$A proof using only a little basic calculus, and not too many cases:
Set
$\alpha(x) = e^{-x}(1 + x); \tag{1}$
then
$\alpha'(x) = -e^{-x}(1 + x) + e^{-x} = -xe^{-x}, \tag{2}$
and
$\alpha(0) = 1; \tag{3}$
we note that
$x > 0 \Rightarrow \alpha'(x) < 0 \tag{4}$
and
$x < 0 \Rightarrow \alpha'(x) > 0 \tag{5}$
with
$\alpha'(0) = 0; \tag{6}$
thus, for $x > 0$,
$\alpha(x) - 1 = \alpha(x) - \alpha(0) = \int_0^x \alpha'(s) ds < 0, \tag{7}$
whence
$e^{-x}(1 + x) = \alpha(x) < 1; \tag{8}$
when $x < 0$,
$1 - \alpha(x) = \int_x^0 \alpha'(s) ds > 0, \tag{9}$
yielding
$e^{-x}(1 + x) = \alpha(x) < 1 \tag{10}$
in this case as well; (8) and (10) together imply
$1 + x < e^x \tag{11}$
when $x \ne 0$; clearly
$1 + 0 = 1 = e^0; \tag{12}$
combining (11) and (12) shows that
$1 + x \le e^x \tag{13}$
for every $x \in \Bbb R$, with strict inequality precisely when $x \ne 0$. QED.
Share Cite Follow edited Mar 14, 2019 at 0:26 J. W. Tanner 63k4 gold badges42 silver badges88 bronze badges answered Aug 19, 2015 at 18:56 Robert LewisRobert Lewis 72.3k5 gold badges63 silver badges122 bronze badges $\endgroup$ Add a comment | 1 $\begingroup$The approximation of the exponential function by its linear Taylor polynomial has remainder term $R(x) := \exp(x) - (1+x)$. The Taylor Remainder Theorem then yields some $\xi$ in between $0$ and $x$ such that $R(x) = \exp''(\xi) x^2 = \exp(\xi) x^2 \ge 0$.
Share Cite Follow answered Nov 4, 2021 at 10:16 community wiki user630227 $\endgroup$ Add a comment | 0 $\begingroup$This answer uses no calculus or geometry.
Prerequisites: Algebra, basic facts about limits and that for $a \gt 0$ we can define $a^x$ for $x \in \Bbb R$ (see limits of rational exponents).
Theorem 1: There is one and only one number $a \gt 0$ satisfying
$$\tag 1 \forall x \in \Bbb R, \; a^x \ge 1 + x$$
Analyzing $\text{(1)}$, you'll be naturally lead to examine
$\tag 2 u_n \le a \le v_n \text{ where } n \ge 2$ with $\tag 3 u_n = (1 + \frac{1}{n})^n \text{ and } \le v_n = (1 - \frac{1}{n})^{-n}$
Searching, you find answer links from this site:
$\quad u_n \text{ is strictly increasing}:\quad$ here $\quad v_n \text{ is algebraically related to } u_n:\quad$ here $\quad u_n \le 3:\quad$ here
By working with the theory in the above links you will conclude that only one real number, call it $e$, can possibly satisfy $\text{(1)}$. Again, as in the first answer link above, you will use the Bernoulli's inequality and
$\tag 4 e^\frac{s}{t} =\lim_{n\to \infty} \Big( 1+\frac{\frac{s}{t}}{\frac{ns}{t}} \Big)^{\frac{ns}{t}}$to wrap things up:
$$ \forall x \in \Bbb R, \; e^x \ge 1 + x$$ Share Cite Follow edited Jun 12, 2020 at 10:38 community wiki 2 revsCopyPasteIt $\endgroup$ Add a comment |You must log in to answer this question.
Not the answer you're looking for? Browse other questions tagged
.- Featured on Meta
- The December 2024 Community Asks Sprint has been moved to March 2025 (and...
- Stack Overflow Jobs is expanding to more countries
Linked
60 How to prove that $\log(x)<x$ when $x>1$? 7 Prove that $e^x\ge x+1$ for all real $x$ 7 To show that $e^x > 1+x$ for any $x\ne 0$ 5 Showing that $e^x > 1+x$ 3 Proof of $e^x - 1 \geq x$ for ${x: -1 \leq x < 0}$ 0 how to prove that $\exp(x)\geq x+1$ 0 Prove that $e^x>x+1 \forall x\ne 0$ 0 How can we show that $e^{-2\lambda t}\lambda^2\le\frac1{e^2t^2}$ for all $\lambda,t\ge0$? 2 How to prove this inequality for exp? 0 Prove the inequality with the mean value/Lagrange theorem See more linked questionsRelated
1 Is most of the GM-AM Inequality in its codicil? 15 Why does the Cauchy-Schwarz inequality hold in any inner product space? 31 Puzzles or short exercises illustrating mathematical problem solving to freshman students 8 Short and intuitive proof that $\left(\frac{n}{k}\right)^k \leq \binom{n}{k}$ 15 which exact integration techniques belong in a first year calculus/analysis course? 18 University-level books focusing on intuition? 11 Inequality for absolute values 0 Interesting real life applications of elementary mathematics 106 What seemingly innocuous results in mathematics require advanced proofs?Hot Network Questions
- Is it possible to get symbolic integral for this?
- How to accept the completion text in PowerShell terminal?
- The third person preterite of "CRIAR" is "crio" without accent instead of "crió¨with accent?
- How to decompose the following rational function into partial fractions?
- Orly airport Metro ticket information
- Handling One-Inflated Count Data Instead of Zero-inflated
- Kids' book where a girl trades her heart for a heart made of lead
- Colombian passport expires in 5 months
- Mama’s cookies too dry to bake
- What is a good way to DM when the party searches for something?
- What religious significance does the fine tuning argument have?
- Review request: evolution of dragon "fire"
- Free Kei Friday
- Why are so many problems linear and how would one solve nonlinear problems?
- How serving documents ensure that a party got the right ones?
- What is this FreeDOS kernel loader found on the “W3x4NTFS” disk image?
- Why does capacitive coupling require a base resistor in an emitter follower?
- Why did my pancake stick to my pan?
- I saw this article online that mentioned animal testing for immersion in water for applications in astronauts. What plans might require this?
- LGPL-like license for 3D models
- What bladed melee weapon would be best suited for a warrior in zero-gravity?
- Search warrants - do the item(s) being searched for limit the scope of the search?
- Did a peaceful reunification of a separatist state ever happen?
- Should I include my legal name on my CV if my preferred name is not reflected on my diplomas? -- Humanities
To subscribe to this RSS feed, copy and paste this URL into your RSS reader.
Từ khóa » Xe^x-1=0
-
Giải X E^x-1=0 | Mathway
-
What Can Say About The Solution Of The Equation Xe^x -1= 0 ... - Quora
-
Solve Xe^x/(e^x-1)-3=0 - Wolfram|Alpha
-
E^x - 1 = 0 How Do I Find X? | Free Math Help Forum
-
Solve The Equation: E^x-1=0 - YouTube
-
Giải Bất Phương Trình Mũ: E^x-1<0 Ai Giải Chi Tiết Giúp Em Với ạ
-
The Area Bounded By Y = X.e^|x| And The Lines |x| = 1, Y = 0 Is - Toppr
-
Exponential Function - Solve $e^x+x=1 - Math Stack Exchange
-
Int0^1(x E^x)/((x+1)^2)dx - Doubtnut
-
[PDF] Random Variables And Probability Distributions
-
[PDF] Actuarial Exam Practice Problem Set 4 - Math